Quantcast
  • Register
PhysicsOverflow is a next-generation academic platform for physicists and astronomers, including a community peer review system and a postgraduate-level discussion forum analogous to MathOverflow.

Welcome to PhysicsOverflow! PhysicsOverflow is an open platform for community peer review and graduate-level Physics discussion.

Please help promote PhysicsOverflow ads elsewhere if you like it.

News

PO is now at the Physics Department of Bielefeld University!

New printer friendly PO pages!

Migration to Bielefeld University was successful!

Please vote for this year's PhysicsOverflow ads!

Please do help out in categorising submissions. Submit a paper to PhysicsOverflow!

... see more

Tools for paper authors

Submit paper
Claim Paper Authorship

Tools for SE users

Search User
Reclaim SE Account
Request Account Merger
Nativise imported posts
Claim post (deleted users)
Import SE post

Users whose questions have been imported from Physics Stack Exchange, Theoretical Physics Stack Exchange, or any other Stack Exchange site are kindly requested to reclaim their account and not to register as a new user.

Public \(\beta\) tools

Report a bug with a feature
Request a new functionality
404 page design
Send feedback

Attributions

(propose a free ad)

Site Statistics

205 submissions , 163 unreviewed
5,047 questions , 2,200 unanswered
5,345 answers , 22,709 comments
1,470 users with positive rep
816 active unimported users
More ...

  Axioms for Euclidean Green's functions's paper by Osterwalder and Schrader

+ 4 like - 0 dislike
1175 views

Hi, I am refering to the following paper http://link.springer.com/article/10.1007%2FBF01645738 (free version from Project Euclid: https://projecteuclid.org/euclid.cmp/1103858969)

on page 91, I don't see how "by (E1), for $f,g\in \mathcal{L}_+$, $(f,U_s(a)g)=(U_s(-a)f,g)$" does this follow from (E1)? where (E1) appears on page 88:(E1) $\mathcal{\sigma}_n(f)=\mathcal{\sigma}_n(f_{(a,R)})$, where $R\in SO(4) , a\in \mathbb{R}^4$.

I appreciate your help.

asked Jun 22, 2015 in Mathematics by MathematicalPhysicist (200 points) [ revision history ]
edited Jun 22, 2015 by Jia Yiyang

2 Answers

+ 3 like - 0 dislike

As @ArnoldNeumaier stated you need to use the definition of (4.3) and (E1). Where (4.3) is

\[(f,g) = \sum_{n,m} G_{n+m}(\theta f_n^* \times g_m)\]

and (E1) states that the Euclidean Green function is invariant under SO(4) rotations and translations. \(G_n(f) = G_n(f_{(\underline{a}, \mathbf{R})})\) where \(f_{(\underline{a}, \mathbf{R})}(\underline{x_1},...,\underline{x_n})= f(\mathbf{R}\underline{x_1} + \underline a,...,\mathbf{R}\underline{x_n} + \underline a)\).

While \(\hat U_s(\vec a)\) generates purely spatial translations on the functions it acts on. Now we can see how the statement follows. 

\[(f,\hat U_s(\vec a)g) = \sum_{n,m} G_{n+m}(\theta f_n^* \times (\hat U_s(\vec a)g)_m) = \sum_{n,m} G_{n+m}(\theta f_n^* \times g_{(-\vec {a}, 1)m}) \]

Then we use (E1) to shift the whole Green function by a value \(\vec a\), where we don't need to worry about \(\theta\) for it is just temporal inversion, which commutes with spatial shifts. Thus we find

\[\sum_{n,m} G_{n+m}(\theta f_n^* \times g_{(-\vec a, 1)m}) = \sum_{n,m} G_{n+m}(\theta f_{(\vec a, 1)n}^* \times g_{m})= \]

\(= \sum_{n,m} G_{n+m}(\theta (\hat U_s(-\vec a)f_{n})^* \times g_{m}) =(\hat U_s(-\vec a)f , g)\)

answered Jun 23, 2015 by Peter Anderson (205 points) [ revision history ]
edited Jul 28, 2015 by Peter Anderson
+ 0 like - 0 dislike

You need to use the definition (4.3) and $U_s(a)^*=U_s(-a)$. (E1) is then applied to the resulting expression.

answered Jun 23, 2015 by Arnold Neumaier (15,787 points) [ no revision ]

Looking how they define \(\hat U_s(a)\) above the statement in question, we should find \(\hat U_s(a)\hat U_s(-a) = 1\) by definition. With the addition of (E1) on the definition (4.3) you then find that it will be Unitary. It would be circular to require from the start that \(U_s(a)^\dagger=U_s(-a)\)

One needs both conditions to derive the desired equation. How one gets the unitarity condition is immaterial. (By the way, there is no circularity if one doesn't use a result to prove thes result itself.)

The whole point of the statements in question is to prove unitary, I'd be very surprised if the required it (as is commented in the paragraph afterwards in the article). I posted how I interpreted it and why I don't think you need that condition. Please correct me if I'm wrong =)
 

@PeterAnderson: There are two different kinds of unitarity involved. Unitarity of the representation of the group on the classical $N$-particle test functions is assumed (and indeed trivial for scalar fields). Unitarity of the group in the Hilbert space of the field theory is the goal. One must make use of the former to get (ultimately) the latter.

Your answer

Please use answers only to (at least partly) answer questions. To comment, discuss, or ask for clarification, leave a comment instead.
To mask links under text, please type your text, highlight it, and click the "link" button. You can then enter your link URL.
Please consult the FAQ for as to how to format your post.
This is the answer box; if you want to write a comment instead, please use the 'add comment' button.
Live preview (may slow down editor)   Preview
Your name to display (optional):
Privacy: Your email address will only be used for sending these notifications.
Anti-spam verification:
If you are a human please identify the position of the character covered by the symbol $\varnothing$ in the following word:
p$\hbar$ysicsOver$\varnothing$low
Then drag the red bullet below over the corresponding character of our banner. When you drop it there, the bullet changes to green (on slow internet connections after a few seconds).
Please complete the anti-spam verification




user contributions licensed under cc by-sa 3.0 with attribution required

Your rights
...